2
$\begingroup$

Can someone help me prove that the limit approaches zero ? I know it does, but I can't prove it.

$$\lim_{n\to\infty}\sum\limits_{k=1}^n\frac{\ln k}{n}\bigg(1-\bigg\{\frac{n}{k}\bigg\}\bigg)\bigg(\frac{1}{2}-\frac{k}{n}\bigg\{\frac{n}{k}\bigg\}\bigg)$$

where $\displaystyle\left\{\frac{n}{k}\right\}$ is the fractional part of $\displaystyle \frac{n}{k}$.

  • 1
    Please consider adding [$\LaTeX$](http://math.stackexchange.com/editing-help#latex) format to your question2012-10-26
  • 0
    how do i do that2012-10-26
  • 0
    @boby I typeset your fomulae using $\LaTeX$, please check if my interpretation is correct. Also to other members, is there a better notation for the Fractional function?2012-10-26
  • 0
    Thanks a bunch, could you help me prove it though? I tested it on wolfram alpha and it aproaches zero quite fast but I still cant prove it2012-10-26
  • 0
    I was just testing the last time sry2012-10-26
  • 0
    You can try to show the efforts you tried. I currently have no idea but surely other people out there do have.2012-10-26
  • 0
    instead of ln(k) the original limit had the Von Mangoldt function in its place, but if I can show either one approachs zero it would be very helpful, I don't know how I would approach something like this, I know there are ways to integrate fractional part functions maybe i could show that the integral approachs zero, but that seems complex on its own2012-10-26
  • 0
    Usually I have seen the fraction part of $x$ denoted by $\{x\}$, obtained by \{x\} in dollar signs, but it has usually been defined, so I am not sure it is standard.2012-10-26
  • 0
    Your "test" had an expression that I took some time to put into $\LaTeX$ which you immediately deleted.2012-10-26
  • 1
    To answer your question "how do i do that": http://meta.math.stackexchange.com/questions/107/faq-for-math-stackexchange/117#1172012-10-26

1 Answers 1